Login

Welcome, Guest. Please login or register.

March 29, 2024, 12:21:19 pm

Author Topic: VCE Physics Question Thread!  (Read 603430 times)  Share 

0 Members and 1 Guest are viewing this topic.

Bestie

  • Victorian
  • Trendsetter
  • **
  • Posts: 130
  • Respect: 0
  • School: Random
Re: VCE Physics Question Thread!
« Reply #675 on: October 31, 2014, 11:16:00 pm »
0
For the vcaa 2013 exam question 1a) the assessors report claims I could use x=ut+1/2at^2 and they got 1.74? I got 1.75 using that method and the report says 1.75 is the wrong ans?

And also...
Q6c) shouldn't there be kinetic energy at points q and p? If not why?

Q15b)was I supposed to give it to two sig fig cause I used 18v which is two sig fig? The assessors gave it to 3 sig fig?

Q15c) how do I know whether to use the RMS voltage or the voltage peak?
« Last Edit: October 31, 2014, 11:57:54 pm by Bestie »

speedy

  • Victorian
  • Forum Obsessive
  • ***
  • Posts: 336
  • Respect: 0
  • School Grad Year: 2014
Re: VCE Physics Question Thread!
« Reply #676 on: October 31, 2014, 11:41:12 pm »
0
Is this seriously an answer that would give full marks?

Spoiler

Physics [50] | Chemistry [45] | English [42] | IT:SD [44]
ATAR: 98.95

lzxnl

  • Victorian
  • ATAR Notes Legend
  • *******
  • Posts: 3432
  • Respect: +215
Re: VCE Physics Question Thread!
« Reply #677 on: October 31, 2014, 11:44:11 pm »
0
Is this seriously an answer that would give full marks?

Spoiler


It seems REALLY brief, but it actually does summarise essentially why Young's model supported the wave theory of light. You should, however, explain why it suggests interference.
2012
Mathematical Methods (50) Chinese SL (45~52)

2013
English Language (50) Chemistry (50) Specialist Mathematics (49~54.9) Physics (49) UMEP Physics (96%) ATAR 99.95

2014-2016: University of Melbourne, Bachelor of Science, Diploma in Mathematical Sciences (Applied Maths)

2017-2018: Master of Science (Applied Mathematics)

2019-2024: PhD, MIT (Applied Mathematics)

Accepting students for VCE tutoring in Maths Methods, Specialist Maths and Physics! (and university maths/physics too) PM for more details

Zealous

  • ATAR Notes Lecturer
  • Victorian
  • Forum Leader
  • ****
  • Posts: 889
  • zeal: great enthusiasm in pursuit of an objective.
  • Respect: +242
Re: VCE Physics Question Thread!
« Reply #678 on: October 31, 2014, 11:48:57 pm »
0
Is this seriously an answer that would give full marks?

Spoiler


I guess you could also talk about diffraction as a wave property?

Quick question: electrons can only make transitions upward (higher energy levels) from ground state right? So if there's n=3 at 4eV and n=4 at 7eV, an electron won't be able to absorb 3eV and take that transition in energy (or it can't go from n=2 straight to ionisation)? Is this because we expect the electron to stabilise by dropping back to ground state first before it can accept any more energy?

Just need some clarification thanks. :D
« Last Edit: October 31, 2014, 11:52:57 pm by Zealous »
vce:
2013: Further [50] (+Premier's) | Methods [48]
2014: Physics [50] | Specialist | Accounting | English Language || ATAR: 99.70 + Australian Student Prize!
uni:
2015: Bachelor of Commerce and Engineering (Honours)

speedy

  • Victorian
  • Forum Obsessive
  • ***
  • Posts: 336
  • Respect: 0
  • School Grad Year: 2014
Re: VCE Physics Question Thread!
« Reply #679 on: October 31, 2014, 11:54:52 pm »
0
I guess you could also talk about diffraction as a wave property?

Quick question: electrons can only make transitions upward (higher energy levels) from ground state right? So if there's n=3 at 4eV and n=4 at 7eV, an electron won't be able to absorb 3eV and take that transition in energy?

Eh, I wouldn't, as VCAA count Young's slit experiment as producing an 'ideal' interference pattern - not a true one. Ie. it doesn't take into account diffraction. (This is from my teacher, who is an assesor). You wouldn't lose marks for it though because it is 'good physics'.

It just seemed crazily simple for 3 marks. I also said what particle model would predict and that bright/dark bands were constructive/destructive interference.



Yeah funny you say that, because the exam I just did (2009 Exam 2 - Q11) assumes an electron is in the 1st excited state for the answer, which then is promoted higher.

Truthfully, it can happen, but it is very unlikely, at this level I have been told to assume it is impossible. But as I just mentioned, this question kinda throws it out.
Physics [50] | Chemistry [45] | English [42] | IT:SD [44]
ATAR: 98.95

speedy

  • Victorian
  • Forum Obsessive
  • ***
  • Posts: 336
  • Respect: 0
  • School Grad Year: 2014
Re: VCE Physics Question Thread!
« Reply #680 on: October 31, 2014, 11:57:03 pm »
0
It seems REALLY brief, but it actually does summarise essentially why Young's model supported the wave theory of light. You should, however, explain why it suggests interference.

Yeah that's what I thought, I definitely wouldn't be this blunt on an exam lol.
Physics [50] | Chemistry [45] | English [42] | IT:SD [44]
ATAR: 98.95

lzxnl

  • Victorian
  • ATAR Notes Legend
  • *******
  • Posts: 3432
  • Respect: +215
Re: VCE Physics Question Thread!
« Reply #681 on: November 01, 2014, 01:41:13 am »
+1
I guess you could also talk about diffraction as a wave property?

Quick question: electrons can only make transitions upward (higher energy levels) from ground state right? So if there's n=3 at 4eV and n=4 at 7eV, an electron won't be able to absorb 3eV and take that transition in energy (or it can't go from n=2 straight to ionisation)? Is this because we expect the electron to stabilise by dropping back to ground state first before it can accept any more energy?

Just need some clarification thanks. :D

Sigh. This gets complicated :P you have all these issues about which energy is it more favourable for it to relax to, or will it preferentially absorb energy...don't worry about that xP

Fluorescence is essentially when excited electrons go via intermediate energy levels back down to the ground state. So to be honest, I don't exactly know how particles determine which energy level to go to. Need to ask a quantum physicist there :P
2012
Mathematical Methods (50) Chinese SL (45~52)

2013
English Language (50) Chemistry (50) Specialist Mathematics (49~54.9) Physics (49) UMEP Physics (96%) ATAR 99.95

2014-2016: University of Melbourne, Bachelor of Science, Diploma in Mathematical Sciences (Applied Maths)

2017-2018: Master of Science (Applied Mathematics)

2019-2024: PhD, MIT (Applied Mathematics)

Accepting students for VCE tutoring in Maths Methods, Specialist Maths and Physics! (and university maths/physics too) PM for more details

Yacoubb

  • Guest
Re: VCE Physics Question Thread!
« Reply #682 on: November 01, 2014, 08:04:16 am »
0
Is this seriously an answer that would give full marks?

Spoiler


For these questions, I mentioned that interference is a typical property of waves. Young's double-slit experiment demonstrated that light interferes constructively and destructively to produce light and dark bands respectively. Thus, Young's experiment supports the wave-model.

speedy

  • Victorian
  • Forum Obsessive
  • ***
  • Posts: 336
  • Respect: 0
  • School Grad Year: 2014
Re: VCE Physics Question Thread!
« Reply #683 on: November 01, 2014, 08:59:12 am »
0

For these questions, I mentioned that interference is a typical property of waves. Young's double-slit experiment demonstrated that light interferes constructively and destructively to produce light and dark bands respectively. Thus, Young's experiment supports the wave-model.

You should also explain why it doesn't support the particle model - for any of these questions, you have to explain why it 'disproves' the other too. (You might have, but just going off what you wrote ^^ :) )
Physics [50] | Chemistry [45] | English [42] | IT:SD [44]
ATAR: 98.95

Yacoubb

  • Guest
Re: VCE Physics Question Thread!
« Reply #684 on: November 01, 2014, 09:01:25 am »
0
You should also explain why it doesn't support the particle model - for any of these questions, you have to explain why it 'disproves' the other too. (You might have, but just going off what you wrote ^^ :) )

Yeah no you're right. On my cheat sheet I've got how X supports the wave-model but not particle, and vice versa. :) Definitely important in fully answering the question!

Yacoubb

  • Guest
Re: VCE Physics Question Thread!
« Reply #685 on: November 01, 2014, 10:48:17 am »
0
Not sure if this has been mentioned yet, but VCAA don't provide super detailed solutions (enough to get the full 3 marks) to worded questions because everyone would just copy it down onto their cheat sheet.

So for a 3 mark question, would we be expected to say for instance:
1 mark - whether it supports the wave or particle model
1 mark - why it supports the particle model (or vice versa)
1 mark - why it doesn't support the wave model (or vice versa)

??

lzxnl

  • Victorian
  • ATAR Notes Legend
  • *******
  • Posts: 3432
  • Respect: +215
Re: VCE Physics Question Thread!
« Reply #686 on: November 01, 2014, 10:55:20 am »
0
Not sure if this has been mentioned yet, but VCAA don't provide super detailed solutions (enough to get the full 3 marks) to worded questions because everyone would just copy it down onto their cheat sheet.

Easiest fix ever: remove the bloody cheat sheet and make people actually remember stuff. Like, what, 99% of the other subjects?
2012
Mathematical Methods (50) Chinese SL (45~52)

2013
English Language (50) Chemistry (50) Specialist Mathematics (49~54.9) Physics (49) UMEP Physics (96%) ATAR 99.95

2014-2016: University of Melbourne, Bachelor of Science, Diploma in Mathematical Sciences (Applied Maths)

2017-2018: Master of Science (Applied Mathematics)

2019-2024: PhD, MIT (Applied Mathematics)

Accepting students for VCE tutoring in Maths Methods, Specialist Maths and Physics! (and university maths/physics too) PM for more details

silverpixeli

  • ATAR Notes Lecturer
  • Victorian
  • Forum Leader
  • ****
  • Posts: 855
  • Respect: +110
Re: VCE Physics Question Thread!
« Reply #687 on: November 01, 2014, 10:59:52 am »
+2
Quick question: electrons can only make transitions upward (higher energy levels) from ground state right? So if there's n=3 at 4eV and n=4 at 7eV, an electron won't be able to absorb 3eV and take that transition in energy (or it can't go from n=2 straight to ionisation)? Is this because we expect the electron to stabilise by dropping back to ground state first before it can accept any more energy?

Just need some clarification thanks. :D

Sigh. This gets complicated :P you have all these issues about which energy is it more favourable for it to relax to, or will it preferentially absorb energy...don't worry about that xP

Fluorescence is essentially when excited electrons go via intermediate energy levels back down to the ground state. So to be honest, I don't exactly know how particles determine which energy level to go to. Need to ask a quantum physicist there :P

This is a question I had all of last year! My physics teacher explained it by saying that the electrons are not excited for long enough for there to be any significant chance for a collision with another photon (unless you have a loooot more photons coming in) and then my first year lecturer mentioned that they're only out of ground state for a few nanoseconds.

So my understanding is that an already-excited electron COULD accept another photon with the right energy to promote it higher (or ionise) but the chance of that collision occurring is just really really slim.

As for which levels they fall back down to, as lzxnl said, that's beyond the scope of the course and you just need to know that it can fall down to anything below it on its way back to ground state!

EDIT: I actually JUST learned how to determine whether a particular path downwards towards n=1 ground state is allowed! Including a simple-english explanation for anyone interested.
Spoiler
It's to do with electron orbitals, which, at a particular energy level, basically 'point' different ways, and you can only move to one which points in a similar way to you. At ground state, there's only a single orbital and it points in direction '0' so if you're at (n=6) and pointing in the maximum direction, '5', you can't reach ground straight away you'd need to go to (n=5) where there's a '4' direction, etc. But if you're in (n=6) and you're only pointing in direction '1', you can jump straight down into (n=1)'s direction '0'.

If you want to learn more, look up orbitals after exams are done and once you understand those, look up the selection rule for emission and absorption!
« Last Edit: November 02, 2014, 01:19:37 am by silverpixeli »
ATAR 99.80 :: Methods [50] | Physics [50+Premier's] | Specialist [47] | Software [48] | English [42] | Legal [39 '12]
+ Australian Student Prize

ATAR Notes Specialist/Methods/Physics Lecturer
ATAR Notes Specialist Maths Webinar Presenter

silverpixeli

  • ATAR Notes Lecturer
  • Victorian
  • Forum Leader
  • ****
  • Posts: 855
  • Respect: +110
Re: VCE Physics Question Thread!
« Reply #688 on: November 01, 2014, 11:10:21 am »
+1
Easiest fix ever: remove the bloody cheat sheet and make people actually remember stuff. Like, what, 99% of the other subjects?

While probably more fair, I think this would have a negative effect (certainly in the year the change occurred) because people would spend time memorising formulas and answers by rote instead and that would take away from solving physics problems. I do agree that the provided formula sheet should be all you need in a physics exam, and if the change was made then they could improve the current one. But it's not really an issue, I mean you're going to do better if you have some intuition behind a concept than if you 'just have it on your cheat sheet' for obvious reasons.

So for a 3 mark question, would we be expected to say for instance:
1 mark - whether it supports the wave or particle model
1 mark - why it supports the particle model (or vice versa)
1 mark - why it doesn't support the wave model (or vice versa)

??

Looks good to me, I'd say;

The experiment supports the wave model for light, as the pattern that is observed on the screen can be explained by diffraction and interference, both wave phenomena. It contradicts the particle model, which does not account for diffraction or interference, instead predicting two bright bands of light directly behind the slits.

Since the question seems to be geared towards a comparison, I'd chuck in a quick diagram of the two expectations if I had time! That's probably more than needed for 3/3, but it can't hurt to give more than required (as long as it's all correct)
ATAR 99.80 :: Methods [50] | Physics [50+Premier's] | Specialist [47] | Software [48] | English [42] | Legal [39 '12]
+ Australian Student Prize

ATAR Notes Specialist/Methods/Physics Lecturer
ATAR Notes Specialist Maths Webinar Presenter

Yacoubb

  • Guest
Re: VCE Physics Question Thread!
« Reply #689 on: November 01, 2014, 12:18:13 pm »
0
Perfect. :)

Thank you :)

Quote
Looks good to me, I'd say;

The experiment supports the wave model for light, as the pattern that is observed on the screen can be explained by diffraction and interference, both wave phenomena. It contradicts the particle model, which does not account for diffraction or interference, instead predicting two bright bands of light directly behind the slits.

Since the question seems to be geared towards a comparison, I'd chuck in a quick diagram of the two expectations if I had time! That's probably more than needed for 3/3, but it can't hurt to give more than required (as long as it's all correct)

Yeah so would that be sufficient to mention how the wave-model is supported? Mentioning that observations of diffraction/interference are consistent with waves, supporting the wave-model, and then describing how the particle-model would predict only two bright bands behind the slits?

I just want to be sure how much to put in, not omitting anything important :)

Also, are incandescent globes on the course? I've been doing pre-2006 exams and in the sections for Light and Matter, it's got questions on incandescent lights and I'm not sure whether or not they're still on the course. :/ thanks!